IMO Shortlist 1995 problem A6


Kvaliteta:
  Avg: 0,0
Težina:
  Avg: 8,0
Dodao/la: arhiva
2. travnja 2012.
LaTeX PDF
Let n be an integer,n \geq 3. Let x_1, x_2, \ldots, x_n be real numbers such that x_i < x_{i+1} for 1 \leq i \leq n - 1. Prove that

\frac{n(n-1)}{2}\sum_{i < j}x_{i}x_{j}>\left(\sum^{n-1}_{i=1}(n-i)\cdot x_{i}\right)\cdot\left(\sum^{n}_{j=2}(j-1)\cdot x_{j}\right)
Izvor: Međunarodna matematička olimpijada, shortlist 1995